LSAT and Law School Admissions Forum

Get expert LSAT preparation and law school admissions advice from PowerScore Test Preparation.

 Administrator
PowerScore Staff
  • PowerScore Staff
  • Posts: 8919
  • Joined: Feb 02, 2011
|
#32027
Please post below with any questions!
User avatar
 bebeg3168
  • Posts: 22
  • Joined: Aug 01, 2022
|
#97963
Hi,

On this question I had:

- R - Q
U - P <
- S - T

I had P going to R and S which satisfied rule 3. U - P satisfied the 4th rule along with R - Q
The answer then could have been either T or Q correct? Answer choice C states Q precisely so that is the answer. I was just wondering about T. This game was different, but in a good way. Any clarification would be wonderful.
User avatar
 Jeff Wren
PowerScore Staff
  • PowerScore Staff
  • Posts: 385
  • Joined: Oct 19, 2022
|
#97986
Hi Bebeg,

If I understand your question correctly, you are asking if Answer (C) would still be the correct answer choice if it had stated "T did not transmit the virus to any other computer on the network" rather than "Q did not transmit the virus to any other computer on the network." The short answer is no.

When answering a logic games questions, it is always important to keep in mind what MUST be true vs. what COULD be true. In your question, you mention that you had "U-P" which satisfied the rules, but is that the only possible way to satisfy that rule? No, T-P also satisfies the rule and is possible in the question. Since the question asks for what MUST be true, Answer (C) would NOT be correct if it had stated "T did not transmit the virus to any other computer on the network" as that statement is not necessarily true.

To effectively tackle this question, we start with the new condition given in the question stem, that "P is the only computer that transmitted to the virus to two other computers." In that case, we know that P MUST be the computer that transmits the virus to R and S, which you had. At this point, looking at rule 5, that "Either T or U transmitted the virus to P," each of those options looks possible here, so we should test out each option.

Option 1
If T goes to P, then P goes to R and S. At this point, we have Q and U still to place. Q must go behind R because the virus can only transmit to Q from R or T (Rule 4), but T has already transmitted the virus once (and can't transmit twice in this question). Finally, U must go behind S because S must transmit the virus to exactly one computer and U is the only computer left.

Option 2
If U goes to P, then P goes to R and S. At this point, we have T and Q still to place. Here, T must go behind S because someone has to go behind S from Rule 2 and Q cannot connect to S from Rule 4. At this point, Q can either connect to R or T, as either is possible from Rule 4.

In either option, Q never transmits the virus to another computer, which is why Answer (C) is correct.

Get the most out of your LSAT Prep Plus subscription.

Analyze and track your performance with our Testing and Analytics Package.